Unraveling the Mysteries of Taylor's Theorem

  • Thread starter Oblio
  • Start date
  • Tags
    Theorem
In summary, the taylor's theorem states that, for any reasonable function f(x), the value of f at a point (x+\delta) can be expressed as an infinite series involving f and its derivatives at the point of x: f(x+\delta) = f(x) + f '(x)\delta + \frac{1}{2!}f ''(x)\delta^{2} + ...
  • #1
Oblio
398
0
Taylor's theorem states that, for any reasonable function f(x), the value of f at a point (x+[tex]\delta[/tex]) can be expressed as an infinite series involving f and its derivatives at the point of x:

f(x+[tex]\delta[/tex]) = f(x) + f '(x)[tex]\delta[/tex] + [tex]\frac{1}{2!}[/tex]f ''(x)[tex]\delta[/tex][tex]^{2}[/tex] + ...

where the primes denote successive derivatives of f(x). (Depending on the function this series may converge for any increment [tex]\delta[/tex] or only for values of [tex]\delta[/tex] less then some nonzero 'radius of convergence'.) This theorem is enormously useful, especially for small values of [tex]\delta[/tex], when the first one or two terms of the series are often an excellent approximation. Find the taylor's series for ln(1+[tex]\delta[/tex]). Do the same for cos [tex]\delta[/tex].


We just started Taylor's theorems now, and it seems like there are many of them. Why exactly are they good and what purpose do they serve?

Is finding the Taylor's series for ln (1+[tex]\delta[/tex]) a matter of substituting it into the above equation or is there more to it?

Thanks again!
 
Last edited:
Physics news on Phys.org
  • #2
In lots of problems, you may have to find f(x+delta) - f(x)... where delta is very small... your calculator might only give you a rounded figure of 0 because the numbers are too close together...

However if you take the taylor series of f(x+delta)... then subtract f(x)... you get: f '(x)delta + f ''(x)delta^2. This will give a good approximation of f(x+delta) - f(x).

That's just one example of why taylor series are useful. I'm sure there are many more.

Is finding the Taylor's series for ln (1+ delta) a matter of substituting it into the above equation or is there more to it?

Yeah, just substitute into the equation, using x = 1. That should work.
 
  • #3
Effectively solving for delta?
 
  • #4
Oblio said:
Effectively solving for delta?

I don't understand what you mean.
 
  • #5
When everything is subbed in, am I solving for delta?
 
  • #6
Oblio said:
When everything is subbed in, am I solving for delta?

No... you'll just be getting a polynomial formula for ln(1+delta)... in terms of delta...
 
  • #7
If I'm subbing x=1, my ln (1+delta) goes where in the formula?
 
  • #8
Oblio said:
If I'm subbing x=1, my ln (1+delta) goes where in the formula?

ln (x + delta) is the left hand side... right hand side is f(x) + f'(x)delta... where f(x) = ln(x)

ln(x + delta)

ln(x+delta) = ln(x) + (1/x)delta + ...

So ln(1+delta) = ln(1) + (1/1)delta + ...
 
  • #9
So ln(1+delta) = ln(1) + (1/1)delta + ...

is the bolded delta in the denominator?
 
  • #10
Oblio said:
So ln(1+delta) = ln(1) + (1/1)delta + ...

is the bolded delta in the denominator?

No. I'm using the formula from your original post exactly... delta doesn't go in the denominator...

I got the 1/1... because the deriative of lnx = 1/x... so f'(1) = 1/1
 
  • #11
Ok, just making sure I was reading it right.

Is that as far as you can go? Since the 2nd derivative would be 0?
 
  • #12
Oblio said:
Ok, just making sure I was reading it right.

Is that as far as you can go? Since the 2nd derivative would be 0?

How do you get 0?

f(x) = ln(x)

f'(x) = 1/x

f''(x) = -1/x^2

So f''(1) = -1 not zero...
 
  • #13
Oh, sub afterwards.
I was deriving 1/1.
 
  • #14
I assume I don't need to do all that many though?
 
  • #15
Is this far enough to do the problem?

ln (x+[tex]\delta[/tex]) = ln(x) + [tex]\frac{1}{x}[/tex][tex]\delta[/tex] + ([tex]\frac{1}{2!}[/tex])[tex]\frac{1}{x^{2}}[/tex]([tex]\delta^{2}[/tex])
 
  • #16
Oblio said:
Is this far enough to do the problem?

ln (x+[tex]\delta[/tex]) = ln(x) + [tex]\frac{1}{x}[/tex][tex]\delta[/tex] + ([tex]\frac{1}{2!}[/tex])[tex]\frac{1}{x^{2}}[/tex]([tex]\delta^{2}[/tex])

Actually I think the question wants you to keep going... and find a general formula for the entire series... do you see a pattern to the terms of ln(1+delta) ?
 
  • #17
denominator and delta are increasing by powers of 1. along with that 1/2!, and its denominator increasing..
 
  • #18
Oblio said:
Is this far enough to do the problem?

ln (x+[tex]\delta[/tex]) = ln(x) + [tex]\frac{1}{x}[/tex][tex]\delta[/tex] + ([tex]\frac{1}{2!}[/tex])[tex]\frac{1}{x^{2}}[/tex]([tex]\delta^{2}[/tex])

careful... it needs to be (-1/x^2) for the third term...
 
  • #19
... i can't say i see a pattern
 
  • #20
Oblio said:
denominator and delta are increasing by powers of 1. along with that 1/2!, and its denominator increasing..

yes, and also, you should have alternating signs...

f''(x) = -1/x^2

f'''(x) = 2/x^3

I recommend finding the first 4 or 5 terms of the series... then you should see the pattern...
 
  • #21
except for the first 2 though? (alternating signs)
and the pattern will boil down to some formula?
 
  • #22
Oblio said:
except for the first 2 though? (alternating signs)
and the pattern will boil down to some formula?

yeah, do 5 terms altogether... get the formula for ln(x+delta) for 5 terms.
 
  • #23
I looked up the higher derivatives for ln x and only found to 4...

ln(x+[tex]\delta[/tex]) = ln (x) + [tex]\frac{1}{x}[/tex][tex]\delta[/tex] + [tex]\frac{1}{2!}[/tex] [tex]\frac{-1}{x^{2}}[/tex][tex]\delta^{2}[/tex] + [tex]\frac{1}{3!}[/tex][tex]\frac{2}{x^{3}}[/tex][tex]\delta^{3}[/tex] + [tex]\frac{1}{4!}[/tex] [tex]\frac{-4}{x^{6}}[/tex][tex]\delta[/tex][tex]^{4}[/tex] + ...
 
  • #24
The question asked for the taylor's series, I need to do more you think?
 
  • #25
Oblio said:
I looked up the higher derivatives for ln x and only found to 4...

ln(x+[tex]\delta[/tex]) = ln (x) + [tex]\frac{1}{x}[/tex][tex]\delta[/tex] + [tex]\frac{1}{2!}[/tex] [tex]\frac{-1}{x^{2}}[/tex][tex]\delta^{2}[/tex] + [tex]\frac{1}{3!}[/tex][tex]\frac{2}{x^{3}}[/tex][tex]\delta^{3}[/tex] + [tex]\frac{1}{4!}[/tex] [tex]\frac{-4}{x^{6}}[/tex][tex]\delta[/tex][tex]^{4}[/tex] + ...

The last term is wrong... it should be -6/x^4... yeah, that's enough terms... now simplify the numbers... ie get rid of the factorials...
 
Last edited:
  • #28
-1, 2, -6 is correct in the pattern?
 
  • #29
Oblio said:
-1, 2, -6 is correct in the pattern?

write out the terms of the series after multiplying out the factorials and simplifying...
 
  • #30
I'm sure this sounds ridiculous but these series are new to me. Multiply OUT the factorials?
 
  • #31
Oblio said:
I'm sure this sounds ridiculous but these series are new to me. Multiply OUT the factorials?

3! = 3*2*1 = 6.

4! = 4*3*2*1 = 24. etc.
 
  • #32
You can multiply what is being added I see. lol
 
  • #33
I need a little example I think in starting the simplification... (Pleeease)
 
  • #34
Oblio said:
I need a little example I think in starting the simplification... (Pleeease)

for example the [tex]\frac{1}{3!}\frac{2}{x^3}(\delta)^3[/tex] becomes [tex]\frac{2}{3*2*1*x^3}(\delta)^3 = \frac{2}{6x^3}(\delta)^3 = \frac{1}{3x^3}(\delta)^3[/tex]

can you do the same type of thing will all the terms, and show the series like that?
 
  • #35
Also meaning that the first two terms cannot be simplified yet?

So, taking [tex]\frac{1}{4!}[/tex] [tex]\frac{-6}{x^{4}}[/tex][tex]\delta^{4}[/tex]

becomes:
[tex]\frac{-6}{4*3*2*1*x^{4}}[/tex][tex]\delta[/tex][tex]^{4}[/tex]

= [tex]\frac{-1}{4x^{4}}[/tex][tex]\delta[/tex][tex]^{4}[/tex] ?
 

Similar threads

  • Differential Equations
Replies
1
Views
759
Replies
1
Views
909
Replies
3
Views
1K
Replies
11
Views
2K
Replies
24
Views
2K
  • Calculus and Beyond Homework Help
Replies
7
Views
255
Replies
11
Views
970
Replies
7
Views
1K
  • Linear and Abstract Algebra
Replies
5
Views
2K
Back
Top